special metal hangers or stirrips called joist hangers are used when joist must be _____ the bottom of the grider or beam

Answers

Answer 1

Answer:

Special metal hangers or stirrups called joist hangers are used when joist must be flush with the bottom of the girder or beam.

Explanation:

A joist hanger also known as a beam hanger is a mechanical device which is used to fasten joists and rafters.The rafters are the carried members to beams and headers are the carrying members.

Thus, special metal hangers or stirrups called joist hangers are used when joist must be flush with the bottom of the girder or beam.

Learn more about construction here:

https://brainly.com/question/14428327

#SPJ4


Related Questions

If an electron has an uncertainty in its velocity of 1.40 m/s, what is the uncertainty (in meters) in its position

Answers

Uncertainty in position:

If the electron has an uncertainty in its velocity of 1.40 m/s then the uncertainty in its position is [tex]4.14\times10^{-4} \text{ m}[/tex].

Heisenberg's uncertainty principle to calculate the required:

Step-1:

We have to apply Heisenberg's uncertainty principle to calculate the uncertainty in the position of an electron. According to the principle:

[tex]$\Delta x \Delta p \geq \frac{h}{4 \pi}$[/tex]

Here,

[tex]$\Delta x$[/tex] is the uncertainty on the position measurement

[tex]$\Delta p$[/tex] is the uncertainty on the momentum measurement

h is the Planck constant.

It is known that the momentum of a particle is calculated as, the product of the mass of the particle and its velocity.

Therefore,

p=mv

Thus the Heisenberg principle becomes:

[tex]$m \Delta x \Delta v \geq \frac{h}{4 \pi}$[/tex]

Here [tex]$\Delta v$[/tex] is the uncertainty in the velocity measurement.

It is given that, [tex]$\Delta v$[/tex]=1.40 m/s. The particle is electron here, thus the mass of the particle m=[tex]9.1 \cdot 10^{-31} \mathrm{~kg}[/tex] and the value of the Plank's constant is, h=[tex]6.62 \times 10^{-34} \mathrm{~m}^{2} \mathrm{~kg} / \mathrm{s}[/tex]

Step-2:

Substituting the values into the equation to get the value of the position uncertainty.

[tex]\Delta x \geq \frac{h}{4 \pi\times m\times \Delta v}\\\geq\frac{6.62\times10^{-34}}{4\pi \times 9.1\times10^{-31}\times1.40} \text{ m}\\\geq 4.14\times10^{-4} \text{ m}[/tex]

To know more about the uncertainty in position, refer to:

https://brainly.com/question/9574825

#SPJ4

Answer the following questions.
1. What is the velocity of a car that traveled a total of 75 kilometers north in 1.5 hours?
2. What is the velocity of a plane that traveled 3,000 miles from New York to California in 5.0 hours?
3. John took 45 minutes to bicycle to his grandmother's house, a total of four kilometers. What was his speed in km/hr?.
4. It took 3.5 hours for a train to travel the distance between two cities at a speed of 120 miles/hr. How many miles lie between the two cities?​

Answers

3.if time=45mins,D=4km,2700secs,4000m then. S=d/t S=1.4m/s

4. An applied force of 6.2 N acts on a 2.1-kg object, pushing it horizontally across a surface
where the coefficient of kinetic friction is 0.15.
a. Draw a free body diagram for the body
b. Determine the frictional force acting.
c. Determine the net force acting on the body
d. Determine the object's acceleration.
Step by step, thank you
[2C]
[2A]
[2A]
[2A]

Answers

The frictional force on the body is  0.25 N the net force on the body is  5.95 N while the acceleration produced is 2.83 m/s^2

What is the net force?

The net force is used to describe the resultant force that acts on the object.

The frictional force that acts on the object s obtained from;

Ff = 0.15 * 2.1-kg * 9.8 m/s^2

Ff = 0.25 N

The net force that acts on the body = F - Ff where F is the applied the force

=  6.2 N - 0.25 N

Net force = 5.95 N

Now;

Net force = ma

a= Net force /m

a = 5.95 N/2.1-kg

a = 2.83 m/s^2

Learn more about the net force:https://brainly.com/question/18031889

#SPJ1

A block of mass 60kg measures 6cm by 5cm by 4cm. Calculate i) The maximum pressure it can exert. The minimum pressure. ii) A block of mass 60kg measures 6cm by 5cm by 4cm . Calculate i ) The maximum pressure it can exert . The minimum pressure . ii )​

Answers

Answer:

4.9 is the answer

Explanation:

pressure is force divided by area

force = mass x acceleration due to gravity

= 60kg x 9.8 m/s

area = length x width x height

=6 x 5 x 4

= 120

pressure = 588/120

= 4.9 pascal

The correct answer for the value of (i) Maximum pressure is [tex]24.5 \dfrac{N}{cm^2}[/tex](ii)Minimum pressure is [tex]19.6 \dfrac{N}{cm^2}[/tex].

Given:

Weight of the block:  [tex]60[/tex] kg

Dimensions of the box:  [tex]6 *4* 5[/tex]

Value of gravitational constant:  [tex]9.8[/tex] m/s²

Now,

[tex]Pressure = \dfrac{Force}{Area}[/tex]

[tex]Force = mass*accelaration[/tex]

[tex]= 60 * 9.8[/tex]

[tex]= 588 N[/tex]

Minimum pressure is when area is maximum:

[tex]= \dfrac{588}{6*5}\\\\= 19.6 \dfrac{N}{cm^2}[/tex]

Maximum pressure when area is minimum:

[tex]= \dfrac{588}{6*4} \\\\= 24.5 \dfrac{N}{cm^2}[/tex]

Maximum and minimum pressure are   [tex]24.5 \dfrac{N}{cm^2}[/tex]  and  [tex]19.6 \dfrac{N}{cm^2}[/tex] respectively.

Learn more about Pressure here:

https://brainly.com/question/21173163

#SPJ7

How does light travel? (1 Point)
A) "Light can only travel in a vacuum and the speed varies depending on whether the source is moving or stationary"
B) "Light can travel in the air or a vacuum and the speed is constant whether the light source is stationary or moving"
C) "Light only travels in a medium and light travels when a sound source is stationary"
D) "Light only travels in a vacuum and travels faster when the sound source is moving"

Answers

Light can travel in the air or a vacuum and the speed is constant whether the light source is stationary or moving which is denoted as option B.

What is Light?

This is referred to the portion of the electromagnetic spectrum which is perceived by the human eyes.

It has a constant speed and can travel in an air or vacuum. This theefrore makes it the most appropriate choice ion this scenario.

Read more about Light here https://brainly.com/question/1363382

#SPJ1

You look in the sky and see two jetliners that you know are of equal size, yet one appears to be much larger. Because of your knowledge of __________, you will assume that the smaller jetliner is farther away

Answers

Answer:

Because of the knowledge of relative size, it will be assumed that the smaller jetliner is farther away.

Explanation:

According to the theory of relative size, the distance that an object has to the viewing individual affects the perception of the individual regarding the size of the object.

As stated in this case, one of the jetliners is farther away from the other. Therefore, even if the jets are of equal size, the one that is at a greater distance is perceived to be smaller as it is at a greater viewing range. The one that is nearer to the individual seems bigger in comparison to the one farther away due to a closer viewing range.

Therefore, the jet that is nearer appears larger.

To know more about relative size, refer to:

https://brainly.com/question/19998265

#SPJ4

A 500 N weight is hung at the middle of a rope attached to two buildings at the same level. If the breaks in the tension exceed 1800 N, what is the minimum angle the rope can make with the horizontal? Group of answer choices 4o 8o 11o 18o

Answers

Answer:

The correct choice is 8o.

Explanation:

The given weight is hanging at the mid point of the rope and the buildings are at the same level, Obtain the given equation by equating the vertical components of force,

2Tsin(φ)=W

where φ is the angle made by rope with the horizontal.

Given T=1800 N and W=500 N, the value of φ is calculated as follows,

2*1800*sin(φ)=500

sin(φ)=0.1388

φ=7.90

φ≈8o

Check out similar questions.

brainly.com/question/13735944

#SPJ10

A glass optical fiber is used to transport a light ray across a long distance. The fiber has an index of refraction of 1.510 and is submerged in water, which has an index of refraction of 1.333. What is the critical angle (in degrees) for the light ray to remain inside the fiber

Answers

The answer is 73.13°.

According to snell's law,

n1sinθi = n2sinθr

n1/n2 = sinθr/sinθi

The critical angle is the angle of incidence at the denser medium when the angle of incidence at the less dense medium is 90°

This means i=C and r = 90°

The Snell's law formula will become

n1/n2 = sinC/sin90°

n2/n1 = 1/sinC

Where n1 is the refractive index of the less dense medium = 1.473

n2 is the refractive index of the denser medium = 1.540

Substituting the values in the formula,

1.540/1.473 = 1/sinC

1.045 = 1/sinC

SinC = 1/1.045

SinC = 0.957

C = sin^-1(0.957)

C = 73.13°

The refractive index of glass is 1.5. It way that the velocity of mild in glass is 1.5 times slower than the rate of mild in a vacuum, the speed of light in glass isn't always unbiased of the shade of mild.

Refractive index is likewise the same as the velocity of light c of a given wavelength in an empty area divided with the aid of its speed v in a substance or n = c/v.

The index of refraction of fabric is a ratio that compares the velocity of light in a vacuum ( c=3.00x108ms ) to the velocity of mild in that precise medium. Because the index of refraction increases, the amount that the material bends the mild increases.

Learn more about the index of refraction here https://brainly.com/question/12469161

#SPJ4

The critical angle (in degrees) for the light ray to remain inside the fiber is 73.13°.

According to snell's law,

n1sinθi = n2sinθr

n1/n2 = sinθr/sinθi

The critical angle is the angle of incidence at the denser medium when the angle of incidence at the less dense medium is 90°.

This means i=C and r = 90°

The Snell's law formula will become

n1/n2 = sinC/sin90°

n2/n1 = 1/sinC

Where n1 is the refractive index of the less dense medium = 1.473

n2 is the refractive index of the denser medium = 1.540

Substituting the values in the formula,

1.540/1.473 = 1/sinC

1.045 = 1/sinC

SinC = 1/1.045

SinC = 0.957

C = sin^-1(0.957)

C = 73.13°

Learn more about critical angle here https://brainly.com/question/15009181

#SPJ4

. A man of mass 80 kg stands in a lift, what is the reaction from the floor of the lift if the lift;A. moves upwards with steady speed. moves upwards with acceleration 0.5m/sC. moves downward with acceleration 0.4m/s(g - 9.8m/s')​

Answers

The reaction of the floor is as follows:

R = 784 NR = 816 N

What is the reaction from the floor for a man in a lift?

The reaction of the floor on the man is equal to the weight of the man and the upward force.

Reaction, R = mg + ma

A)  When the lift moves upwards with steady speed.

R = 80 * 9.8 + * 80 * 0

R = 784 N

B) When the lift moves upwards with acceleration 0.4m/s

R = 80 * 9.8 + * 80 * 0.4

R = 816 N

In conclusion, the reaction of the floor changes as the body accelerates upwards.

Learn more about reaction and weight at: https://brainly.com/question/25555136

#SPJ1

If the strength of the earth's electric field is 100 N/CN/C , how does the magnitude of the electric force on the droplet compare to the weight force

Answers

Magnitude of electric force = 15 x 10⁻¹⁰ N

Magnitude of weight force = 64.092 x 10⁻⁶ N

Given

Diameter of rain drop = 2.5mm

Radius = 1.25 mm = 1.25 × 10⁻³ m/sec

Drop has a charge = +15pC

15pC = 15×10⁻¹²C

Electric field (E) = 100 N/C

Electric force on the charge

= charge x electric field = qE

= 15 x 10⁻¹² x 100

= 15 x 10⁻¹⁰ N.

This force will act in upward direction as the force on the positive charge will always acts in the direction of the electric field.

Volume of rain droplet(V) = 4/3 π R³

Volume V = 4/3 x 3.14 x ( 1.25 x 10⁻³)³

                 = 6.54 × 10⁻⁹m³

As Density of water = 1000 kg / m³

Density =Mass/Volume

So, Mass of rain droplet = 1000 x 6.54 x 10⁻⁹ kg

= 6.54 x 10⁻⁶ kg .

Now, Weight(W) = mg

= 6.54 x 10⁻⁶ x 9.8

= 64.092 x 10⁻⁶ N.

As the electric force is 15 × 10⁻¹⁰ N

So, weight force is more than the electric force.

Hence, the magnitude of the electric force on the droplet is less as compare to the weight force.

Learn more about Electric force & weight here https://brainly.com/question/13258339

#SPJ4

(picture attached) use grass method
4. A block of unknown substance is submerged in water. A light ray in the water strikes the angle of 16°, what is the index of refraction of the unknown substance? (nwater = 1.33) substance at an angle of 45° from the normal. If the refracted ray in the substance is at an 5. If a light ray passes from a substance with low index of refraction to another substance with high index of refraction, will the ray bend away from or closer to the normal? Why? 6. What is the angle of refraction? n₁=1 24° n₂ = 1.8​

Answers

Answer:

Explanation:

Step 1

1 of 2

\textbf{Given}Given

n_{w}=1.33n

w

=1.33

$\theta_w=31\text{ ^\circ

}$

$\theta_b=27\text{ ^\circ

}$

\textbf{Approach}Approach

In this problem we are going to use Snell's law.

\textbf{Solution}Solution

The definition of Snell's law of refraction is

\begin{align} {n_1\cdot\sin \theta_1}={n_2\cdot \sin \theta_2} \end{align}

n

1

⋅sinθ

1

=n

2

⋅sinθ

2

 

where indexes 11 and 22 represent two different mediums. Since the motion is in the water we write

\begin{align} &{n_{w}\cdot\sin\theta_{w}}={n_{b}\cdot\sin\theta_{b}} \\ &{n_b}={n_{w}\cdot \frac{\sin\theta_{w}}{ \sin \theta_{b}} } \\ &{n_b}={1.33\cdot \frac{\sin31^\circ}{ \sin27^\circ} } \\ &\boxed{{n_b}=1.5} \end{align}

 

n

w

⋅sinθ

w

=n

b

⋅sinθ

b

n

b

=n

w

sinθ

b

sinθ

w

n

b

=1.33⋅

sin27

sin31

n

b

=1.5

After rubbing on the apparatus, your pith pall became negatively charged. What type of charge does the apparaTapus now have

Answers

         The apparatus must be Positively Charged.

Charge interaction:

    As from the charge interaction we came to know that when we   rub two objects together then there will be three possibility.

        1. opposite charge objects attract each other

        2. like charge object repel each other

        3. neutral object with any charges will attract each other.

    These happens due to friction generated on rubbing of object which in reverse create electron and get transfer.

  So, In the question it said that after rubbing on apparatus pith ball

   become negative charge which means it has gained the electron.

          Thus, the apparatus must have lose electron and would be

          positively charged.

   

          Learn more about pith ball here:

                     https://brainly.com/question/1131516

                       #SPJ4

 

       

Two objects, m1 = 0.6 kg and m2 = 4.4 kg undergo a one-dimensional head-on collision
Their initial velocities along the one-dimension path are vi1 = 32.4 m/s [right] and vi2 = 8.6 m/s [left].
The two objects stick together after the perfectly inelastic collision
a) Calculate the velocity after the collision.
b) Determine how much kinetic energy is lost due to the collision

Answers

Explanation:

Hello !

look at the attachment above ☝️ and if you have any questions your welcome.

a) The velocity after the collision.is 11.456 m/s.

b) The kinetic energy lost due to the collision is 44.564 J.

What is conservation of momentum principle?

When two bodies of different masses move together each other and have head on collision, they travel to same or different direction after collision.

The external force is not acting here, so the initial momentum is equal to the final momentum. For inelastic collision, final velocity is the common velocity for both the bodies.

m₁u₁ +m₂u₂ =(m₁ +m₂) v

Given are the two objects, m1 = 0.6 kg and m2 = 4.4 kg undergo a one-dimensional head-on collision. Their initial velocities along the one-dimension path are vi1 = 32.4 m/s [right] and vi2 = 8.6 m/s [left].

(a) Substitute the values, then the final velocity will be

0.6 x32.4 +4.4 x 8.6 = (0.6+4.4)v

v = 11.456 m/s

Thus, the velocity after collision is 11.456 m/s.

(b) Kinetic energy lost due to collision will be the difference between the kinetic energy before and after collision.

= [1/2m₁u₁² +1/2m₂u₂² ] - [1/2(m₁ +m₂) v²]

Substitute the value, we have

= [1/2 x 0.6 x32.4² + 1/2 x4.4 x 8.6²] - [1/2 x(0.6+4.4)11.456²]

= 44.564 J

Thus, the kinetic energy lost due to the collision is 44.564 J.

Learn more about conservation of momentum principle

https://brainly.com/question/14033058

#SPJ2

A speed skater moving across frictionless ice at 8.0 m/s hits a 5.0-m-wide patch of rough ice. She slows steadily, then continues on at 6.0 m/s. What is her acceleration on the rough patch

Answers

Her acceleration on the rough patch is 2.8m/s².

To find the answer, we need to know about the Newton's equation of motion.

What are the Newton's equation of motion?

These are as follows

V= U +atS = Ut+1/2 at²V²-U²= 2aS

V= Final velocity

U = initial velocity

t= time

S= distance

What will be acceleration if initial velocity, final velocity and distance are 8 m/s , 6 m/s and 5m respectively?Here U= 8 m/s, V = 6 m/s and S = 5mSo, 6²-8²= 2a× 5=10a

36-64 = 10a

-28= 10 a

a= -28/10= -2.8 m/s²

Negative sign indicates the decrease of acceleration.

Thus, we can conclude that the acceleration on the patch is 2.8 m/s².

Learn more about the Newton's equation of motion here:

brainly.com/question/8898885

#SPJ4

Without using afood flask, suggest another ways would you keep food warm a till to the hospital

Answers

Another way to keep food warm till you arrive at the hospital is by putting the food in a foil and wrapping it up with a towel to retain the heat.

What is Food preservation?

Food preservation is defined as the process by which a cooked food or process food is kept in such a way that microorganisms cannot affect their taste and texture.

To prevent a cooked food from getting cold, you can put the food in an aluminium foil and wrap it with a towel to prevent heat loss.

Learn more about food preservation here:

https://brainly.com/question/16972821

#SPJ1

If you place a 3.5-kg object on a spring which has a spring constant 144-N/m, and stretch the spring so the object starts oscillating. How much time does it take to complete one cycle

Answers

Answer:

M = 3.5 kg         mass of object

F = - K x     where K is the spring constant

K = 144 N/m

ω = (K/M)^1/2     for simple harmonic motiom

ω = (144 kg/s^2 / 3.5 kg)^1/2 = 6.41 / sec

f =  ω / (2 * π)  = 1 / P

P = (2 Π / ω) = 2 * 3.14 / 6.41 sec

P = .98 sec

Check: f = 1 / P = 1.02 /sec

ω = 2 * π * f = 2 * 3.14 * 1.02 = 6.41 sec

Calculate the frequency of the 3rd harmonic of a 0.5 meter string with a linear density of 0.03 kg/m at a tension of 50 newtons.

Answers

Frequency of third harmonic is 122.48 HZ

Theory:

Frequency of third harmonic with length L , linear density μ , Tension

T is given as

                  fₙ= n v/2L

                   fₙ = n/2L × √T/√μ

      plugging in the values here-

                   fₙ =  3/ 2×0.5 × √50 / √0.03

                   fₙ = 122.48  Hz

          Thus, the frequency of third harmonic is 122.48 HZ

        Learn more about frequency here:

               https://brainly.com/question/14888403

                      #SPJ4    

Which arrow represents the substance’s change of state?

Answers

Answer:

Its P path describes the change of state i.e from solid to liquid.

What is solid ?

"A solid is that state of matter which has a fixed shape, mass, and volume. It suffers very small changes in volume by changing the temperature. It can not be compressed, e.g. — Sand, Wood, Copper, Ice, etc."

What is liquid ?

"A liquid is a sample of matter that conforms to the shape of a container in which it is held, and which acquires a defined surface in the presence of gravity. The term liquid is also used in reference to the state, or condition, of matter having this property."

A series combination of a 48.0-mF capacitor and a resistor are connected to a 12-V battery. After one second, the voltage across the capacitor is 10 V. What is the resistance of the resistor

Answers

The resistance of the resistor is .

Given:-  A series combination of a 48.0-mF capacitor and a resistor are connected to a  battery. After one second, the voltage across the capacitor is .

To Find:- We have to find the value of the resistance of the resistor.

By using the concept of the resistance it will be solved.

According to the problem,

The value of the resistance of the resistor is .

#SPJ4

If the radio waves transmitted by a radio station have a frequency of 79.5 MHz, what is the wavelength of the waves, in meters

Answers

If the radio waves transmitted by a radio station have a frequency of 79.5 MHz, the wavelength of the waves, in meters is 3.77m.

What are radio waves?The electromagnetic spectrum's longest wavelengths, which are found in radio waves, are normally found at frequencies of 300 gigahertz. They may be reflected and refracted to change direction, and they do not harm the human body if they are absorbed by it. They are perfect for communicating due to their characteristics.

What are Frequency and wavelength?When describing the temporal rate of change seen in oscillatory and periodic phenomena like mechanical vibrations, radio waves, and light, frequency is a crucial parameter utilized in science and engineering.The length of a waveform signal that is propagating in space or over a wire is measured by the separation between identical points (adjacent crests) in the adjacent cycles. Frequency, or the number of wave cycles per second, is inversely related to wavelength.

Given: Velocity of light,  c = 3.00 x 10⁸ m/s  m/s

Frequency, f = 7.95 x 10⁷/s

For wavelength, the required formula is

[tex]λ = c/ f \\λ = 3 * 10 ^{8} / 7.95 * 10^{7} \\λ = 3.77[/tex]

Hence, the required wavelength is 3.77 meters.

To learn more about frequency and wavelength refer to:

https://brainly.com/question/12924624

#SPJ4

A box with a mass of 2 kg only has four forces acting on it: One force of 16 N due East. One force of 24 N due South. One force of 16 N due West. One force of 18 N due North. What is the magnitude of the box's acceleration, in m/s2

Answers

Answer:

The acceleration of the object is 3 m/s2.

Explanation:

The force on the object is acting along the four directions which are perpendicular to each other. Since the north and south directions are opposite to each other, the magnitude of the net force along north-south direction is (24-18)=6 N.Since the east and west directions are opposite to each other, the magnitude of the net force along east-west direction is (16-16)=0 N.Therefore the magnitude of the net force on the object is 6 NBy Newton's second law, the force F on the object is given by the formula F=ma. Here m= 2kg, therefore 6 =(2)a which gives a = 3 m/s2.        

Learn more about Newton's law.

brainly.com/question/13447525

#SPJ10

What is the current flowing through the circuit shown? (V= 120 V, R₁ = 20 02,
R₂= 50 Q, R3= 1022) (Ohm's law: V = IR)
R₁
ww
ww
R₂
ww
R₂

Answers

Assumed that resistors are connected in series .

R_net=20+50+10=80ohm

Voltage=V=120V

Current=I

V/R120/803/21.5A

Using the information from the previous problem (A 2kg ball rotates on the end of a 1.4m long string. The ball makes 5 revolutions in 4.4s.), calculate the centripetal force on the ball.
Group of answer choices

A) 140.9N

B) 1.1N

C) 110.4N

D) 5.7N

Answers

The centripetal force on the ball is 140.9 N; option A

What is the centripetal force on the ball?

The centripetal force on the ball is given by the formula below:

F = mv²/r

where;

m is massv is linear velocity r is radius

The linear velocity, v = wr

v = 5 * 2π/4.4 * 1.4 = 9.93 m/s

F = 10² * 2/1.4 = 140.9 N

In conclusion, the centripetal force is derived from the mass, velocity and radius of the path.

Learn more about centripetal force at: https://brainly.com/question/20905151

#SPJ1

The density of a certain type of plastic is 0.77 g/cm3. If a sheet of this plastic is 10.0 m long, 1.0 m wide, and 1 cm thick, what is its mass

Answers

Mass is 77,000 g

Given

The density of a certain type of plastic = 0.77 [tex]$\mathrm{g} / \mathrm{cm}^{3}$[/tex]

Length = 10.0 m = 1000 cm

Width = 1 m = 100 cm

Thickness = 1 cm

Formula

Volume = length x width x thickness

Mass = Density x Volume

Explanation

Density

Density is how much "stuff" is contained in a specific quantity of space is determined by its density. For instance, a block of the harder, lighter element gold (Au) will be denser than a block of the heavier element lead (Pb) (Au). Styrofoam blocks are less dense than bricks. Mass per unit volume serves as its definition.

Volume V = 1000 x 100 x 1

             V = 100,000 [tex]cm^{2}[/tex]

Mass M= 0.77 [tex]$\mathrm{g} / \mathrm{cm}^{3}$[/tex] x 100,000 [tex]cm^{2}[/tex]

M = 77,000 g

So, Mass is 77,000 g

Learn more about this problem here:

https://brainly.com/question/598902

#SPJ4

If the red and green balls fall off opposite sides of the table, the green ball will make a louder
sound. why?
select one:
o a. the green ball has more potential energy, meaning it had less kinetic energy.
potential energy equals sound energy.
o b. the green ball has more mass, so it had more kinetic energy. the kinetic energy
transferred to sound energy.
o c. the green ball was moving at a faster rate, and speed of movement determines the
loudness of the sound.
od. the red ball has less potential energy, so the green ball had more potential energy.
as energy transferred from one ball to the other, sound was created.

Answers

B. The green ball has more mass, so it had more kinetic energy, the kinetic energy transferred to sound energy.

What is kinetic energy?

This is the energy possessed by a moving object, and it depends on mass and speed of the object.

K.E = 0.5mv²

where;

m is mass of the objectv is the speed of the object

Thus, we can conclude that, the green ball has more mass, so it had more kinetic energy, the kinetic energy transferred to sound energy.

Learn more about kinetic energy here: https://brainly.com/question/25959744

#SPJ1

calculate the kinetic energy of 200kg object that is moving with a speed 15ms.

Answers

Answer:

See below

Explanation:

Kinetic Energy = 1/2 * m * v^2

                        = 1/2 * 200 * (15)^2 = ________ joules

The kinetic energy of the 200 kg object moving with a speed of 15 m/s is 11250 Joules (J).

Given:

Mass of the object (m) = 200 kg

Speed of the object (v) = 15 m/s

For the kinetic energy (KE) of an object, we use the formula:

KE = (1/2) × m × v²

where:

KE is the kinetic energy,

m is the mass of the object,

v is the speed of the object.

The kinetic energy of the object:

KE = (1/2) × 200 × (15 )²

KE = (1/2) × 200 × 225

KE = 11250

KE = 11250 Joules (J)

Therefore, the kinetic energy of the 200 kg object moving with a speed of 15 m/s is 11250 Joules (J).

To know more about kinetic energy:

https://brainly.com/question/30107920?referrer=searchResults

#SPJ3

_____________is the addition of wave energy as waves interact, producing larger waves and ____________ is the subtraction of wave energy as waves interact, producing smaller waves.

Answers

Constructive interference is the addition of wave energy as waves interact, producing larger waves and destructive interference is the subtraction of wave energy as waves interact, producing smaller waves.

To find the answer, we need to know about the interference of waves.

What's the interference of waves?Interference of waves is the result of superposition of waves (transverse or longitudinal) at a certain place. Interference is of two typesConstructive interferenceConstructive interferenceDestructive interferenceHow are the constructive and destructive interference formed?When two waves are superimposed within the phase, then constructive interference pattern is formed.When two waves are superimposed with out of phase, destructive interference pattern is found.

Thus, we can conclude that the constructive interference is the addition of wave energy and destructive interference is the subtraction of wave energy.

Learn more about the interference of waves here:

brainly.com/question/12466679

#SPJ4

While standing at the top of an 100 m high observatory, you accidentally dropped your phone through the grates (it is now falling straight down to the ground). a. What is the velocity of the phone after 4.0 s? b. How far does the phone fall during this time? C. Will your phone hit the ground? If not, how long more before it hits the ground?​

Answers

Using the idea of motion under gravity, it will take 0.5 seconds more before the phone hits the ground.

What is the distance covered?

The distance covered is obtained form the equations of kinematics under gravity.

Now;

v = u + gt

Recall that it was dropped from a height hence u = 0 m/s

v = gt

v = 9.8 m/s^2 * 4 s

v = 39.2 m/s

Now;

h = ut + 1/2gt^2

but u = 0 m/s

h = 1/2gt^2

h = 1/2 * 9.8 * (4)^2

h = 78.4 m

The phone  will not hit the ground within this time

For the phone to hit the ground;

h = 1/2gt^2

if h = 100 m

100 = 1/2 * 9.8 * t^2

2 * 100/9.8 =  t^2

t = √2 * 100/9.8

t = 4.5 seconds

It will take about 0.5 seconds more before the phone will hit the ground.

Learn more about motion under gravity:https://brainly.com/question/15120445

#SPJ1

from the same height (and at the same time), one ball is dropped and another ball is fired horizontally. which one will hit the ground first

Answers

Both balls will take the same amount of time to hit the ground if they are dropped and fired from the same height.

What is Projectile motion:

It is the motion of an object thrown or projected into the air, subject to only the acceleration of gravity. The object is called a projectile, and its path is called its trajectory.

For a ball fired horizontally from same height and time, there is only a vertical acceleration on it towards the ground.

This acceleration is equal to the acceleration due to gravity (g = 9.81 m/s^2).

A ball dropped from the same height will also experience the same vertical acceleration downwards as the horizontal acceleration.

Therefore,

Both balls will take the same amount of time to hit the ground if they are dropped and fired from the same height.

Learn more about Projectile motion here:

https://brainly.com/question/11049671

#SPJ4

a ray of light incident on a mirror, at an angle of 45°. Another mirror is placed at an angle of 45° to the first ones as shown. Sketch the patch of the ray until it emerges from the two mirrors (2mks) 45° www.www 45° ​

Answers

Answer:

If the ray of light is deflected by 45 degrees by the first mirror its total deflection by mirror (I) is 90 deg. (incident = 45 and exit ray equals 45 deg)

The second mirror will cause a net deflection of 90 degrees and the total deflection will be 180 deg or in opposite  direction to the  incident ray.

Other Questions
Avoidant children, feeling that emotional expression can be dangerous, become responsible for their own emotion regulation without help from a caregiver. In extreme cases, these children provide emotional care for their caregiver. What term identifies this pattern What is Mittani laws FOR 50 POINTS!In great detail, describe how to solve the advanced function below.4^(8 - 2x) = 256I understand the solution to the problem is (2). I would like a detailed description of how exactly to solve this problem. Event A has a probability of occurring of 4/5 . Event B has a probability of occurring of 1/3. If events A and B are independent of one another, what is the probability of both events occurring? Use the properties of 30-60-90 and 45-45-90 triangles to solve for x in each of the problems below. Then decode the secret message by matching the answer with the corresponding letter/symbol from the exercises. a student collects 0.36 mol of methane gas (CH4) at STP. Determine the following mass of the methane gas A museum tour guide who prepares an interviewing guide for a group viewing paintings in one room is likely to use a _____. Write an equation that represents the line.Use exact numbers. What were the two main reasons that the british granted independence to india? You are pouring canned sodas Into a cylindrical pitcher. Each canof soda is 12 cm tall and has a diameter of 6.5 cm. The pitcher is36 cm tall and has a diameter of 20 cm. How many cans of sodawill the pitcher hold? The function f(x) = three-fourths(10)x is reflected across the x-axis to create the function g(x). which ordered pair is on g(x)? David is making a rectangular exercise yard for his turtle, Ralph. The perimeter of the yard is16x + 12x +183x2x + 6x +93xinches in total. The length of the yard measuresinches. Find an expression to represent the measure of the width in inches A circular swimming pool has a diameter of 8 m and a depth of 2 m. What is the volume of the swimming pool? we can tell that side DE is parallel to side....A) ABB) We can't be sure it's parallel to any side Using the Addition Method, solve for x in the following system of linear equations.2y + x = 4y - 3x = 2a.) x = 1b.) x = 2c.) x = 8d.) x = 0 Qu sabe usted y a quin conoce?B. El restaurante El CanalCompleta el dilogo con la forma correcta de los verbos entre parntesis.MARCO ANTIONIO: (T: Saber) dnde est el restaurante El Canal?JAVIER: Cmo no! Ceci y yo (conocer) muy bien a los dueos.MARCO ANTIONIO: Yo (conocer) a su hija Luca, pero no (saber) la direccin del restaurante. Luca dice que sus padres (saber) cocinar mejor que nadie.JAVIER: Ceci (saber) su nmero de telfono si quieres llamar. Debes(conocer) a toda la familia. Todos son muy simpticos. How were Homer's poems passed down?A. Word of mouthB. Carved stone tabletsC. Wood carvingsD. Scrolls made of papyrus Simplify (-8ab) (3ab). a) -24a7b b) -5a7b c) -5a26 d) -24a26 Josh is 72 inches tall. Ruben is 9/10 as tall as josh. John is 11/12 tall as ruben. a. what is rubens height in inches b. what fractional of of joshs height is johns? Cheryl is responsible for cybersecurity at a mid-sized insurance company. She has decided to use a different vendor for a network antimalware than she uses for host antimalware. Is this a recommended action, why or why not